ECE Gate Question Paper - 2020

You might also like

Download as pdf or txt
Download as pdf or txt
You are on page 1of 33

Hyderabad | Delhi | Ahmedabad | Pune | Bhubaneswar | Bengaluru | Lucknow | Chennai | V ayawada | Vizag | Tirupathi | Kukatpally | Kolkata

GATE-2020
Questions with Detailed Solutions

ELECTRONICS &
COMMUNICATION ENGINEERING

ACE Engineering Academy has taken utmost care in preparing the GATE-2020
Examination Solutions. Discrepancies, if any, may please be brought to our notice.
ACE Engineering Academy does not owe any responsibility for any damage or loss
to any person on account of error or omission in these solutions. ACE Engineering
Academy is always in the fore front of serving the students, irrespective of the
examination type (GATE/ESE/PSUs/PSC/GENCO/TRANSCO etc.,).

All Queries related to GATE-2020 Solutions are to be sent to the following email address
hyderabad@aceenggacademy.com | Contact Us : 040 - 23234418, 19, 20

www.aceenggacademy.com
GATE - 2020
Electronics & Communication Engg. 02/02/20
Question with Detailed Solutions

SUBJECTWISE WEIGHTAGE
1 MARK 2 MARKS
S. No. NAME OF THE SUBJECT Total Marks
QUESTIONS QUESTIONS

1 Verbal Ability 2 3 8

2 Numerical Ability 3 2 7

3 Engineering Mathematics 5 3 11

4 Network Theory 3 2 7

5 Control Systems 2 3 8

6 Digital Circuits & Microprocessors 3 3 9

7 Signals & System 2 3 8

8 EDC & VLSI 2 4 10

9 Analog Circuits 3 5 13

10 EMT 3 4 11

11 Communication Systems 2 3 8

Total No. of Marks 30 35 100

ACE Engineering Publications Hyderabad  Delhi  Pune  Bhubaneswar  Bengaluru  Lucknow  Chennai  Vijayawada  Vizag  Tirupati  Kolkata  Ahmedabad
2 Electronics & Communication Engineering

Section : General Aptitude who may have difficulties in repaying loans, and
its arises because of excess liquidity following the
01. A superadditive function f(.) satisfies the following East Asian crisis.
property Which one of the following sequences show the
f(x1 + x2) ≥ f(x1) + f(x2) correct precedence as per the given passage?
Which of the following functions is a superadditive (a) Banking crisis — Sub-prime lending crisis —
function for x > 1 ? Global financial crisis — East Asian crisis
1
(a) x (b) ex (b) East Asian crisis — Sub-prime landing crisis
(c) e−x (d) x — Banking crisis — Global financial crisis
01. Ans: (b) (c) Sub-prime lending crisis — Global financial
Sol: am is super additive function, if a > 1. crisis — Banking crisis — East Asian crisis
example 25 > 22 + 23 (d) Global financial crisis — East Asian crisis —
f(x1 + x2) > f(x1) + f(x2) Banking crisis — Sub-prime landing crisis
So, ex is super additive fucntion. ( a e > 1) 03. Ans: (b)
02. The Canadian constitution requires equal
04. a,b,c are real numbers. The quadratic equation
importance be given to English and French. Last
ax2 − bx + c = 0 has equal roots, which is β, then
year, Air Canada lost a lawsuit, and had to pay a
(a) β2 = ac (b) β = b/a
six- figure fine to French-speaking couple after
(c) b2 ≠ 4ac (d) β3 = bc / (2a2)
they filed a complaints about formal in-flight
04. Ans: (d)
announcements in English lasting 15 seconds,
Sol: ax2 − bx + c = 0
as compared to informal 5 seconds messages in
Sum of roots =     ba & 2  ba - - - - - (1)
French.
The French-speaking couple were upset at ____. Product of roots =  )   ca & 2  ca - - - - - (2)
(a) the in-flight announcement being made in Now (1) × (2) ⇒ (2β)(β2) = (b/a)(c/a)
English. (2β3) = bc/a2
(b) the English announcements being clearer than β3 = bc/(2a2)
French ones
05. A circle with centre O is shown in the figure.
(c) equal importance being given to English and
A rectangle PQRS of maximum possible area is
French.
inscribed in the circle. If the radius of the circle is
(d) the English announcements being longer than
a, then the area of the shaded portion is _____.
French ones.

02. Ans: (d) P Q

O
03. The global financial crisis in 2008 as considered a
to be the most serious world-wide financial crisis,
which started with the sub-prime landing crisis S R
in USA in 2007. The sub-prime lending crisis led
to the banking crisis in 2008 with the collapse of Find shaded area
Lehman Brothers in 2008. The sub-prime lending (a) πa2 – 3a2 (b) πa2 − 2a2
refers to the provision of loans to those borrowers (c) πa2 − a2 (d) ra2 - 2 a2
ACE Engineering Publications Hyderabad  Delhi  Pune  Bhubaneswar  Bengaluru  Lucknow  Chennai  Vijayawada  Vizag  Tirupati  Kolkata  Ahmedabad
3 GATE_2020_Questions with Solutions

05. Ans: (b) 09. The following figure shows the data of students
Sol: Area of shaded portion enrolled in 5 years (2014 t0 2018) for two schools
= Area of circle - Area of rectangle P and Q. During this period, the ratio of the average
Area of rectangle is maximum, when it becomes number of the students enrolled in school P to the
square. average of the difference of the number of students
Q enrolled in schools P and Q is ____
a 9
Schoole P

Number of students (in thousands)


a O 8 Schoole Q
7
S 6

5
Diagonal = 2a
4
Area of square = 12 (diagonal) 2 = 2a2
3
2
` Area of shaded portion
1
= Area of circle - Area of square
= πa2 − 2a2 0 2014 2015 2016 2017 2018
Year
06. Select the word that fits the analogy: (a) 8 : 23 (b) 31 : 23
Explicit: Implicit:: Express: _____ (c) 23 : 8 (d) 23 : 31
(a) Repress (b) Impress 09. Ans: (c)
(c) Suppress (d) compress Sol: Average number of students enrolled in school P
06. Ans: (c) + + + 6000 + 4000
in 5 years = 3000 5000 5000 5
= 4600
07. The untimely loss of life is a cause of serious Average number of students enrolled in school Q
global concern as thousands of people get killed + + + 7000 + 5000
in 5 years = 4000 7000 8000 5
____ accidents every year while many other die = 6200
____ diseases like cardio vascular disease, cancer, Average of the difference of the number of students
etc. in school P and Q = 6200 - 4600 = 1600
(a) from, of 4600 = 23
Desired ratio = 1600 8
(b) from, from 23 : 8
(c) during, from
(d) in, of 10. It is quarter past three in your watch. The angle
07. Ans: (d) between the hour hand and the minute hand is ___
(a) 0° (b) 7.5°
08. He was not only accused of theft ___ of conspiracy. (c) 22.5° (d) 15°
(a) rather than 10. Ans: (b)
(b) rather Sol: Time: 03 : 15
(c) but also Angle between hands  30h  112m
(d) but even Here, h = 3 and m = 15
08. Ans: (c) So,  30 # 3  165  7.5 o
2
ACE Engineering Publications Hyderabad  Delhi  Pune  Bhubaneswar  Bengaluru  Lucknow  Chennai  Vijayawada  Vizag  Tirupati  Kolkata  Ahmedabad
4 Electronics & Communication Engineering

Section : Electronics & Communication Circle of radius 1 with centre at (0, 0)


Engineering indicates pure induction and capacitive load.

01. The current in the RL-circuit shown below is 03. For a vector field A , which one of the following is
i(t) = 10 cos(5t − π/4)A. FALSE ?
The value of the inductor (rounded off to two (a) A is irrotational if d2 A = 0
decimal places) is ____ H. (b) d # ]d # A g = d ]d.A g - d2 A
R i(t) (c) d # A is also another vector field
(d) A is solenoidal if d.A = 0
03. Ans: (a)
+ Sol: For a vector field Av
200cos(5t) V L v is irrotational (or) conservative only
− (a) A
if d # A v =0

(b) d # ^d # A v h  d ^d.Av h  d2 A
v

is used to obtain EM wave equation.
01. Ans: 2.828
(c) curl of a vector, Av is an another vector field
Sol: V = 200+0 o
i.e. d # Av is another vector field.
I  10+  45 o
v =0
v is said to be solenoidal if d.A
(d) A
V
Z  I  20+  45 o Therefore option (a) is FALSE

Z=14.14+j14.14

04. A 10-bit D/A converter is calibrated over the
XL full range from 0 to 10V. If the input to the D/A
converter is 13A (in hex), the output (rounded off
14.14  14.14
XL = wL = 14.14 ⇒ L   5 to three decimal places) is ____ V.
L = 2.828H 04. Ans: 3.066
02. The impedances Z = jX, for all X in the range Sol: DAC output = Number of steps for given digital
(−∞, ∞), map to the Smith chart as input × step size
(a) a circle of radius 1 with centre at (0, 0) Where Number of steps for digital input 13AH is
(b) a point at the centre of the chart. ⇒ 1 × 162 + 3 × 161 + 10 × 160 = 314
10 10
(c) a circle of radius 0.5 with centre at (0.5, 0) step size = 10 = 1024
2
(d) a line passing through the centre of the chart. Thus, DAC output
02. Ans: (a) 10
=
= 314 # 1024
3140
=
z = +jx 1024 3.066 volts .
Sol: x = +1
r=0
d2 y dy
05. The general solution of 2  6  
dx dx 9y 0 is
(a) y  C1 e 3x  C2 e -3x
(b) y  ^C1  C2 xh e -3x
x = –1 (c) y = C1 e 3x
z = –jx (d) y  ^C1  C2 xh e 3x

ACE Engineering Publications Hyderabad  Delhi  Pune  Bhubaneswar  Bengaluru  Lucknow  Chennai  Vijayawada  Vizag  Tirupati  Kolkata  Ahmedabad
5 GATE_2020_Questions with Solutions

ACE Engineering Publications Hyderabad  Delhi  Pune  Bhubaneswar  Bengaluru  Lucknow  Chennai  Vijayawada  Vizag  Tirupati  Kolkata  Ahmedabad
6 Electronics & Communication Engineering

05 Ans: (d)
Im
Sol: Given f(D)y = 0, wher f ^Dh  D2  6D  9 s-plane
⇒ Auxiliary equation is f(m) = 0
⇒ m2 − 6m + 9 = 0 ×
⇒ m = 3,3 are real and equal roots
∴ The general solution of given v × × × Re
differential equation is y = (C1 + C2x)e3x.
∴ option (d) is correct. ×
Γ

06. The pole-zero map of a rational function G(s) G(s) - plane


is shown below. When the closed contour Γ is Im
mapped into the G(s)-plane, then the mapping
encircles.
Im
s-plane
Re

Ν=1
× × × Re

×
Γ 07. The output y[n] of a discrete-time system for an
input x[n] is
y 5n? = max x 5k?

(a) the origin of the G(s)-plane once in the -3 # k # n

The unit impulse response of the system is


counter-clockwise direction.
(a) unit step signal u[n]
(b) the point −1+j0 of the G(s)-plane once in the
(b) 1 for all n
counter-clockwise direction.
(c) 0 for all n
(c) the origin of the G(s)-plane once in the
(d) unit impulse signal δ[n]
clockwise direction.
07. Ans: (a)
(d) the point −1+j0 of the G(s)-plane once in the
Sol: y(n) = max |x(k)| −∞ ≤ k ≤ n
clockwise direction.
The unit impulse response is
06. Ans: (c) If we apply x(n) = d(n)
Sol: Once encircles the origin in the clock wise y(n) = max|d(k)| = 1 n ≥ 0
direction. = u(n)

ACE Engineering Publications Hyderabad  Delhi  Pune  Bhubaneswar  Bengaluru  Lucknow  Chennai  Vijayawada  Vizag  Tirupati  Kolkata  Ahmedabad
7 GATE_2020_Questions with Solutions

08. Consider the recombination process via bulk By multiply equation (2) with 3, then we get
traps in a forward biased pn homojunction diode. -3V + 6 VTH = 18_______(2A)
The maximum recombination rate is Umax. If the By adding equation (1) with (2A), then we get
electron and the hole capture cross-sections are 5 VTH = 18
equal, which one of the following is FALSE ? V=
18
=
TH 5 3.6V
(a) Umax depends exponentially on the applied bias
(b) Umax occurs at the edges of the depletion region 10. The two sides of a fair coin are labelled as 0
in the device and 1. The coin is tossed two times independently.
(c) With all other parameters unchanged, Umax Let M and N denote the labels corresponding to the
decreases if the intrinsic carrier density is outcomes of those tosses. For a random variable X,
reduced. defined as X = min(M,N), the expected value E(X)
(d) With all other parameters unchanged, Umax (rounded off to two decimal places) is _____
increases if the thermal velocity of the carrier 10. Ans: 0.25
increases. Sol: Coin tossed two times:
08. Ans: (c) Outcomes X = min (m, N)
Sol: Intrinsic carrier density will not affect generation 0 0 0
and recombination rate. so, statement (c) is false. 0 1 0
1 0 0
09. In the circuit shown below, the Thevenin voltage 1 1 1
VTH is P(X = 0) = 3/4, P(X = 1) = 1/4
2V 2Ω 4Ω = E (X) / =
x i P (X x i)
–+ i
+ 3 1
1Ω 2Ω VTH
E (X)  0 # 4  1 # 4  0.25
1A 2A

11. In the given circuit, the two-port network has the
(a) 2.8 V (b) 4.5 V
impedance matrix 5Z? = = G . The value of ZL
40 60
(c) 3.6 V (d) 2.4 V 60 120
for which maximum power is transferred to the
09. Ans: (c)
load is ____ Ω.
Sol: Nodal Analysis: I1 I2
10 Ω
VTH + +
V1 2V 2Ω 4Ω
–+ 120 V +
– V1 [Z] V2 ZL
+ – –

1A 1Ω 2A 2Ω VTH
– 11. Ans: 48
Sol: V1 = 40I1 + 60I2 ______(1)
V2 = 60I1 + 120I2______(2)
V1 _ V1  VTH  2 i V
1   0 S-I ZL = I 2
1 2 2

3V1 - VTH = 0 _______(1) 10 Ω


I1 I2

VTH _ VTH  V1  2 i +
[Z] + V2
2 0 V1


2 2
-V1 + 2VTH = 6 _______(2) V2
ZL= I
2

ACE Engineering Publications Hyderabad  Delhi  Pune  Bhubaneswar  Bengaluru  Lucknow  Chennai  Vijayawada  Vizag  Tirupati  Kolkata  Ahmedabad
8 Electronics & Communication Engineering

also, 13. A transmission line of length 3λ/4 and having a


10 I1 + V1 = 0_____(3) characteristic impedance of 50Ω is terminated
with a load of 400Ω. The impedance (rounded off
(3) in (1)
to two decimal places) seen at the input end of the
-10I1 = 40 I1 + 60I2
transmission line is ____ Ω
-50I1 = 60 I2 → I1  6 I _______(4)
5 2 13. Ans: 6.25
3
Sol: Given: Length of transmission line: ,  4
(4) in (2) V2  60 ; 56 E I2  120I2

Zo = 50Ω

ZR = 400Ω
V2 = -72 I2 + 120I2 3
As length of the transmission line is 4 ,
V
V2  48I2 & Z L  I 2  48 (odd multiples of quarter wavelengths like
2
λ 3 λ 5λ
4 , 4 , 4 ....) and hence this line must be a
quarter wave transmission line.
12. In the circuit shown below, all the components are Assume this quarter wave transmission line is
ideal. If Vi is +2V, the current I0 sourced by the op- Z2
lossless then Z in = Z o
amp is _____ mA. R
2
= 50

1kΩ 400
= 6.25Ω

1kΩ 5V
– I0 14. The random variable

W ] t g ] t g dt,
3

Vi + 1kΩ Y #
–5V -3


where ] t g  )
1; 5 # t # 7
12. Ans: 6
0; otherwise
Sol:
1kΩ
and W(t) is a real white Gaussian noise process
with two-sided power spectral density SW(f) = 3
+5V W/Hz, for all f. The variance of Y is ____
1kΩ Io 14. Ans: 6

Vo Sol: Y  # W ^ t h ^ t h dt
3

Vi + -3
1kΩ Let φ(t) = h(-(t-t)) ⇒ where t is the symbol
–5V
duration
# W ^ t h h ` _ t  ij dt Let t = t ⇒ dt = dt
3
Y
Given op-amp is in non-inverting mode, then -3

W ^  h h _   t i d
output # 3
Y
Vo  e1  R f o Vin  c1  1k m  2V  4V
R 1k -3

i
Y(t) = W(t) * h(t)
KCL at node Vo ⇒ SY(f) = |H(f)|2 sW(f)
Vo  Vi Vo
  Io  0 2Y = E[Y2] - E[Y]2
1k 1k
E 6Y @ S ^ f h df #
#= H ^ f h s W ^ f h df
3 3
=
2 2
42 4
I o  1k  1k
Y
-3 -3

E 6Y@  E : #- 3 W ^ t h ^ t hD dt
3

Io = 6mA

ACE Engineering Publications Hyderabad  Delhi  Pune  Bhubaneswar  Bengaluru  Lucknow  Chennai  Vijayawada  Vizag  Tirupati  Kolkata  Ahmedabad
9 GATE_2020_Questions with Solutions

 E : # E 7W ^ t hA ^ t h dtD
3 Entropy will be maximum when the probabilities
-3
are equal i.e P(X = +2) = 1/2 = 0.5
= 0

S W ^ f h = 20 17. A single crystal intrinsic semiconductor is at a


N

temperature of 300K with effective density of
= 0 # H ^ f h 2 df
N 3

2 -3 states for holes twice that of electrons. The thermal
 0 #  ^ f h 2 df aa  ^ t h  h ` _ t   ijk
N 3
voltage is 26mV. The intrinsic Fermi level is
2 -3
shifted from mid-bandgap energy level by
|φ(f)|2 = |H(f)|2
(a) 13.45 meV (b) 18.02 meV
^ t h dt _from the rayleigh's energy theorem i
N0

2 #
-3
3 2
(c) 26.90 meV (d) 9.01 meV
= d N0 nEz 17. Ans: (d)
2 Sol: T = 300K, NV = 2 NC VT = 26mV
= 3 # 2 `a E z = 2 j
E C  E V KT
e Co
N
EF  
2 ,n N V
=6 2
EC  EV
if N C  N V then E Fi  ⇒ Midband gap
15. If v1 , v2 ,....., v6 are six vectors in R , which one of
4 2
energy
the following statements is FALSE ?
Given NV = 2NC
(a) These vectors are not linearly independent
E F  E Fi  2 ,n e 2NC o
(b) Any four of these vectors form a basis for R4 KT N

(c) It is not necessary that these vectors span R4
C

(d) If {v1, v3, v5, v6} spans R4, then it forms a basis  E Fi  KT ,n c 1 m
2 2
for R4
15. Ans: (d)  E Fi  0.026 ,n c 1 m
2 2
Sol: option (d) is a wrong statement.
= EFi + 9.01 × 10-3
if {v1, v3, v5, v6} spans R4 then it may (or) may
= 9.01meV
not be a basis for R4.
i.e. To form a basis, the set {v1, v3, v5, v6} should
18. The figure below shows a multiplexer where S1
be linearly independent set.
and S0 are the select lines, I0 to I3 are the input data
lines, EN is the enable line, and F(P, Q, R) is the
16. A binary random variable X takes the value +2 or
output F is
−2. The probability P(X = +2) = α. The value of α
0 EN
(rounded off to one decimal place), for which the
R Ι0
entropy of X is maximum, is ____ F
0 Ι1 MUX
16. Ans:0.5 Ι2
fx(x) R
Sol: 1 Ι3 S1 S0

1− α α
P Q

(a) PQ + QR (b) PQR + PQ


x
−2 +2 (c) P + QR (d) Q + PR

ACE Engineering Publications Hyderabad  Delhi  Pune  Bhubaneswar  Bengaluru  Lucknow  Chennai  Vijayawada  Vizag  Tirupati  Kolkata  Ahmedabad
10 Electronics & Communication Engineering

ACE Engineering Publications Hyderabad  Delhi  Pune  Bhubaneswar  Bengaluru  Lucknow  Chennai  Vijayawada  Vizag  Tirupati  Kolkata  Ahmedabad
11 GATE_2020_Questions with Solutions

18. Ans: (a)


fx  2x  e1 - x cos y `  cos y j  z.e 1 + y
2f -1
2

Sol: As EN = 0 → The multiplexer is Enabled.


` _fx i_1,0,ei  e1 - 1 _  cos 0 i  ^ e h .e 1 + 0  1  1  0
-1

The output F  S1 S 0 I 0  S1 S 0 I1  S1 S 0 I2  S1 S 0 I 3
F  P Q.R  P.Q.0  P. Q.R  P.Q.1 Hence, option (b) is correct
= QR + PQ .
21. A digital communication system transmits a block
19. The loop transfer function of a negative feedback of N bits. The probability of error in decoding a
system is bit is α. The error event of each bit is independent
K _s  11 i of the error events of the other bits. The received
G^ s hH^ s h  .
s _s  2 i_s  8 i block is declared erroneous if at least one of its
bits is decoded wrongly. The probability that the
The value of K, for which the system is marginally
received block is erroneous is
stable, is ______.
(a) 1 - αN (b) 1 - (1-α)N
19. Ans:160
(c) αN (d) N(1-α)
Sol: ] g ] g  K ^s  11h
s ^s  2h^s  8h 21. Ans: (b)
G s H s
Sol: 1 2 N
Characteristic equation: 1 + G(s)H(s) = 0 3 ----------

: s(s+2) (s+8) + K(s+11) = 0
P(1 bit error) = α, P(1 bit correct) = 1 - α
: s3 +10s2 + 16s + Ks + 11K = 0
P(N bit error) = αN, P(N bit correct) = (1 - α)N
: s3 + 10s2 + (16+K) s+ 11K = 0
(N bits are independent)
3
s
1 16+K The received block declared erroneous if
atleast one of bits decoding wrongly (i.e correct
s2 10 11K
probability means all bits should be correct)
s1 10(16 + K) − 11K P(received block is erroneous) = 1 - (1 - α)N
10 = 1 - P (all bits correct)
s0 11K
= P (any one bit or more error)

For marginal stability, 22. The components in the circuit shown below are
^  h
c 10 K 16 11K m  0 ideal. If the op-amp is in positive feedback and the
10
10K + 160 - 11K = 0 input voltage Vi is a sine wave of amplitude 1 V,
K = 160 the output voltage V0 is
1k Ω

20. The partial derivative of the function


1V +5V
V
f ^x, y, zh  e1 - x cos y  xze - (1 + y )
1
2
0 Vi +
with respect to x at the point (1, 0, e) is −1V 1k Ω V0

Options
1 –5V
(a) 1 (b) 0 (c) e (d) -1
(a) a square wave of 5V amplitude.
20. Ans: (b)
Sol: Given f _ x, y, z i  e1 - x cos y  xze 1 + y
2
-1 (b) an inverted sine wave of 1V amplitude.
(c) a constant of either +5 V or -5V.
and (x, y, z) = (1, 0, e) (d) a non-inverted sine wave of 2V amplitude.
ACE Engineering Publications Hyderabad  Delhi  Pune  Bhubaneswar  Bengaluru  Lucknow  Chennai  Vijayawada  Vizag  Tirupati  Kolkata  Ahmedabad
12 Electronics & Communication Engineering

22. Ans: (c)


Im
Sol: If Vin > + 5V then Vo changes from −5V to +5V
If Vin < −5V then Vo changes from +5V to −5V 1
3rd order pole
As Vin = ! 1V therefore the input cannot flip the
max

output.
So, output stays at either +5V (or) −5V (b) −1 1
+5V
Re

Vo
+
–5V R1 = 1kΩ
−1

R2 = 1kΩ

+ V
– in
Im

Transfer characteristics 1
3rd order pole
Vo

+5
(c) −1 1
Re

–5 +5 Vin
−1

–5
Im
23. Which one of the following pole-zero plots
1
corresponds to the transfer function of an LTI 4th order pole
system characterized by the input-output difference
equation given below?
(d) −1 1
3
Re
y [n]  / ( 1) k x [n  k]
k=0

Im

1 −1
3rd order pole

(a) −1 1 23. Ans: (c)


Re
Sol: y ]ng  / ] 1gk x ]n  kg
3

k=0

y ]ng  x ]ng  x ]n  1g  x ]n  2g  x ^n  3h
−1 Y ]z g   -1  -2  -3
H ]z g 
X ]z g
1 z z z

ACE Engineering Publications Hyderabad  Delhi  Pune  Bhubaneswar  Bengaluru  Lucknow  Chennai  Vijayawada  Vizag  Tirupati  Kolkata  Ahmedabad
13 GATE_2020_Questions with Solutions

z 3  z2  z  1  ]z  1g]z2  1g 25. In an 8085 microprocessor, the number of address


=
z3 z3 lines required to access a 16K byte memory bank
one zero at z = 1 & 2 zeros at z = ±j is ______.
3poles at z = 0 25. Ans: 14
From the given pole-zero plot answer will be Sol: In 8085 microprocessor, the number of address
option (c) lines required to a access a 16KB memory bank is
2n ≥ 16KB ⇒ 2n ≥ 214 × 8
24. In the circuit shown below, all the components The number of adder lines required are n = 14.
are ideal and the input voltage is sinusoidal. The
magnitude of the steady-state output V0 (rounded 26. For the components in sequential circuit shown
off to two decimal places) is _______ V. below, tpd is the propagation delay, tsetup is the setup
C1=0.1µF D2
time, and thold is the hold time. The maximum clock
+ frequency (rounded off to the nearest integer), at
230 V(rms) D1 C2=0.1µF Vo which given circuit can operate reliably is _____
− MHz.

24. Ans: 650.54


FlipFlop1
= = tpd = 2 ns
Sol: Vin 230
max
# 2 Vm
tpd = 3 ns
tpd = 2 ns

During negative cycle C1 charges [D1 forward tsetup = 5 ns


CLK thold = 1 ns
biased]
VC1
IN
– + – +
VC1 – FlipFlop2
+
Vmsinwt tpd = 8 ns
– tsetup = 4 ns
+ thold = 3 ns

KVL, −Vm + VC1 = 0
VC1 = Vm 26. Ans: 76.92
Sol: In any sequential circuit, the condition for proper
During positive cycle D2 forward biased
operation is
Vm
Clock period (T) ≥ tpd + tcomb logic + tsetup
– + D2
+ + Thus,
Vm Vo
– For Flipflop 1 ⇒ T ≥ 8 + 5 ⇒ T ≥ 13ns

For Flipflop 2 ⇒ T ≥ 3 + 2 + 2 + 4 ⇒ T ≥ 11ns

Thus T ≥ 13ns
KVL, −Vm −Vm + Vo = 0 1
⇒ frequency of operation f #
Vo = 2Vm 13 # 10 -9

= 2=
# 230 # 2 650.530V fmax = 0.07692 × 109
= 650.54V. fmax = 76.92MHz.

ACE Engineering Publications Hyderabad  Delhi  Pune  Bhubaneswar  Bengaluru  Lucknow  Chennai  Vijayawada  Vizag  Tirupati  Kolkata  Ahmedabad
14 Electronics & Communication Engineering

ACE Engineering Publications Hyderabad  Delhi  Pune  Bhubaneswar  Bengaluru  Lucknow  Chennai  Vijayawada  Vizag  Tirupati  Kolkata  Ahmedabad
15 GATE_2020_Questions with Solutions

27. Which one of the following options contains 29. Ans: 0.3
two solutions of the differential equation Sol:
 `y  1jx ? fX(x)
dy
dx
(a) ln|y – 1| = 0.5x2 + C and y = – 1 1/12
(b) ln|y – 1| = 2x + C and y = 1
2

(c) ln|y – 1| = 0.5x2 + C and y = 1 −2 x

(d) ln |y – 1| = 2x2 + C and y = –1 fY(y)

27. Ans: (c) 1/24


Sol: Given dx  ` y  1 j x
dy
y
−10 14

⇒ # y  1 dy  # xdx  c
1

P(Y ≤ 7 / X ≥ 5)
Y ==2XP(Y− 6≤ 7 / Y ≥ 4)
⇒ log ` y  1 j  2  C
x2 X = 5 ⇒ YY#= 74, Y $ 4)
=
P (
P (Y $ 4)
f l^x h

fa dx  log f ^x h  C p
7
# f ^x h # 1/24 dy
P (4 # Y $ 7) 3/24
= = 4
= =
∴ log |y − 1| = (0.5)x2 + C P (Y $ 4) 14 10/24 0.3
# 1/24 dy
The second solution is y = 1 4

Hence, option (c) is correct.


30. For the given circuit, which one of the following is
28. The magnetic field of a uniform plane wave in the correct state equation?
vacuum is given by 0.5H
H v ` x, y, z, t)  (at x  2at y  bat z j cos (t  3x  y  z) .
i
The value of b is _____.
28. Ans: 1 + 0.25F
i1 2Ω v 1Ω i2
Sol: H  ` xt  2yt  bzt j cos `t  3x  y  z j −

H  H o cos _t  k.r i


 H o cos 9t  `  3xt  yt  zt j . ` xxt  yyt  zzt jC

So H o  xt  2yt  bzt d =vG  = 4  4G=vG  =4 0G=i1G
(a) dt
i 2 4 i 0 4 i2
k  3xt  yt  zt
d =vG  = 4  4G=vG  =4 4G=i1G
For plane wave H o .kt = 0 (b) dt
i 2 4 i 4 0 i2
`  3xt  yt  zt j
` xt  2yt  bzt j . 0 d =vG  = 4 4 G=vG  =0 4G=i1G
911 (c) dt
i 2 4 i 4 0 i2
−3 + 2 + b = 0
b=1 d =vG  = 4  4G=vG  =0 4G=i1G
(d) dt
i 2 4 i 4 4 i2
29. If X is a random variable with uniform probability
density function in the interval [– 2, 10]. 30. Ans: (c)
For Y = 2X – 6, the conditional probability Sol: Using Topology:
P (y # 7 | x $ 5) (rounded off to three decimal Inductor → Link
places) is ______. Capacitor → Twigs

ACE Engineering Publications Hyderabad  Delhi  Pune  Bhubaneswar  Bengaluru  Lucknow  Chennai  Vijayawada  Vizag  Tirupati  Kolkata  Ahmedabad
16 Electronics & Communication Engineering

Current sources → Links (c) (–3, –2)


iL (d) (–1, 0)
31. Ans: (d)
+ +
Sol: Characteristic Equation:
V2Ω VC
– – i1Ω i2 s3 + 3s2 + (K + 2)s + 3K = 0
i1
s3 + 3s2 + 2s + Ks + 3K = 0
s(s2 + 3s + 2) + K(s + 3) = 0
By KCL, K (s  3 )
G (s) H (s) 
s ( s  1) ( s  2 )
dV
i L  i Z  C dtC  i1 X _____(1)
By KVL,
di Im
 V2X  L L  VC  0 _____(2)
dt
Now, voltage across 2W, current theory 1W

⇒ V2X  2 8i1  i LB _____(3)


V K=∞ K=0 K=0
i1  22X  i L K=0
× RL × × RL × Re
−3 −2 −1 BP s = 0
1. i1W - VC = 0 ⇒ i1W = VC _____(4)

i L  i2  4 < C F  VC
1 dV

dt
92 _i1  i L iC  1 L  VC  0 Break point exist between (-1, 0)
di
2 dt
1 dVC   
4 dt VC i L i2
32. In a digital communication system, a symbol
1 di L  
2 dt VC 2i L  2i1 S randomly chosen from the set {s1, s2, s3, s4} is
dVC transmitted. It is given that s1 = –3, s2 = –1, s3 =
 4VC  4i L  4i2
dt +1 and s4 = +2. The received symbol is Y = S + W.
di L W is a zero-mean unit-variance Gaussian random
 2VC  4i L  4i1
dt
RS V variable and is independent of S.Pi is the conditional
SS dVC WWW
SS dt WW  4 4 VC probability of symbol error for the maximum
SS di L WW  >  H> i H  >4 0H>i H
0 4 i1

SS W 2 4 L 2 likelihood (ML) decoding when the transmitted
dt W
T X symbol S = si. The index i for which the condional
symbol error probability Pi is the highest is _____.
31. The characteristic equation of a system is 32. Ans: 3
s3 + 3s2 + (K + 2)s + 3K = 0. Sol: S1 = -3, S2 = -1, S3 = +1, S4 = +2
In the root locus plot for the given system, as K
Y=S+W
varies from 0 to ∞, the break-away or break-in 1 - ~ /2
W = N(0, 1) =
2
e
point(s) lie within 2
(a) (–2, –1)
fs1 (s1) &  3  W  S1
(b) (– ∞, –3)

ACE Engineering Publications Hyderabad  Delhi  Pune  Bhubaneswar  Bengaluru  Lucknow  Chennai  Vijayawada  Vizag  Tirupati  Kolkata  Ahmedabad
17 GATE_2020_Questions with Solutions

VI = +15V V0 = 9V
f s (s )
1 1

R 3= 1 k Ω
Decision R 1= 1 k Ω
P(s1) correct Boundaries

−3 x
R2
f s (s )
2 2
Vz = +3.3V
S2 = −1+W


33. Ans: 800
−2 −1 0
f s (s ) Sol: VR2 = 0.7 + 3.3 = 4V
3 3
Vo = IR1 + IR2
P(s3) correct S3 = 1+W 9 =I(R1 + R2) [Given Vo = 9V]
Neglect base current
9  R _1k  R2 i
0 1 1.5 4

2
fs (s )
4 4
R2 = 800Ω
P(s4) correct
S4 = 2+W V0
I R1=1k
1.5 2
s1 s2 s3 s4
+
– +
0.7V VR2 R2 I
+ –
Since all symmbols are equally likely, decision
3.3V
boundaries are mid points. –
i.e., for S1, decision boundary -2 V i.e., anything
below -2 V considered as S1.
For S2 decision boundaries are -2 & 0V, i.e., 34. The state diagram of a sequence detector is shown
any thing -2V to 0V considered as S2. below. State S0 is the initial state of sequence
For S3 decision boundaries are 0V & 1.5V. detector. If the output is 1, then
1/0 0/0 0/0
For S4 decision boundaries is 1.5V i.e., 1.5V to ∞
considered as S4.
P(Correct symbol receiveing is less for S3 only S0 S1 S2 S3
0/0 1/0 0/0
therefore S3 has highest conditional symbol error
probability). 1/0 1/0
0/1
S4
1/0
33. In the voltage regulator shown below, VI is the
unregulated input at 15V. Assume VBE = 0.7V and (a) the sequence 01011 is detected.
the base current is negligible for both the BJTs. (b) the sequence 01001 is detected.
If the regulated output V0 is 9V, the value of R2 is (c) the sequence 01010 is detected.
_______ Ω. (d) the sequence 01110 is detected.
ACE Engineering Publications Hyderabad  Delhi  Pune  Bhubaneswar  Bengaluru  Lucknow  Chennai  Vijayawada  Vizag  Tirupati  Kolkata  Ahmedabad
18 Electronics & Communication Engineering

ACE Engineering Publications Hyderabad  Delhi  Pune  Bhubaneswar  Bengaluru  Lucknow  Chennai  Vijayawada  Vizag  Tirupati  Kolkata  Ahmedabad
19 GATE_2020_Questions with Solutions

34. Ans: (c) short circuited at signal frequencies. The input vi


Sol: Given S0 is initial state is direct coupled. The low frequency voltage gain
For output to be 1, the sequence of states traversed vo/vi of the amplifier is
are VCC= 10V
S0 , S1 , S2 , S3 , S4 , S3 RC
10 kΩ
i.e., input is → 0 1 0 1 0 v0
C1
1 µF RL
10 kΩ
35. A system with transfer function G (s)  (s  1)1(s  a) , vi
RE CE
20 k Ω 100 µF
a > 0 is subjected to an input 5cos3t. The steady
1 VEE = –10V
state output of the system is cos (3t − 1.892) .
10
The value of a is _____.

35. Ans: 4
Sol: G ] s g 
1 (a) - 89.42 (b) - 256.42
^s  1h^s  ah
a 0
(c) - 178.85 (d) - 128.21
x(t) = 5 cos3t
y ss ] t g  cos ^3t  1.892h 36. Ans: (a)
1

10 Sol: DC Analysis: 10V
x(t) = Acos(ω0t + θ)  0.7  _  10V i
IE  RC 10kΩ
H ] s g S = j~  H ] 0g +H ] 0g
20k
9 . 3 V
= 20 = 0.465mA = I C
0

y ss ] t g  A H ] 0g cos ^ 0 t    +H ] 0gh
V DC

IC
=
gm = DC
+
Vt 0.0178 0.7V –
5cos3t 1 yss(t)
G (s ) = 20kΩ
(s + 1)(s + a ) IE RE
–10V
A = 5 ω0 = 3

G ] s g S = j3  1
_ j3  1 i_ j3  a i
AC Analysis:
G ^ j3 h  1 + Vo
a2  9 1  9
Vbe gmVbe Rc RL
From the above expression

A G ]0g 
1

10
5  1 & 5 a2  9 Vo = −gm (RC||RL)Vbe

10 a 92
 10
Vi = Vbe
25 − a2 + 9
a=4 Vo  g m _R C || R L i Vbe
  g m _R C || R L i
Vi Vbe
36. For the BJT in the amplifier shown below, = −0.0178(10k||10k)
VBE = 0.7V, kT/q = 26 mV. Assume that BJT output
= −0.0178(5k)
resistance (ro) is very high and the base current is
negligible. The capacitors are also assumed to be = −89.423

ACE Engineering Publications Hyderabad  Delhi  Pune  Bhubaneswar  Bengaluru  Lucknow  Chennai  Vijayawada  Vizag  Tirupati  Kolkata  Ahmedabad
20 Electronics & Communication Engineering

37. Using the incremental low frequency small-signal 38. A one-sided abrupt pn junction diode has a
model of the MOS device, the Norton equivalent depletion capacitance CD of 50 pF at a reverse
resistance of the folowing circuit is bias of 0.2V. The plot of 1/ C2D versus the applied
VDD voltage V for this diode is a straight line as shown
in the figure below. The slope of the plot is _____
R
# 1020 F -2 V -1 .
gm, rds 2
1/CD


r +R
(a) +ds
1 g r
(b) rds + R
m ds
1
(c) rds + R + g m rds R (d) rds + g m + R
V
37. Ans: (a) 0
(a) -5.7 (b) -0.4
Sol:
+ (c) -1.2 (d) -3.8
Vgs gmVgs
38. Ans: (*) insufficient data
rds
Sol: [If we assume V0 = 0.5V]

q
e A D o
N N
CD 
rds
(D)
2 _ V0  VR i A N D
N
(G)
+ 1
C2D 
Vgs gmVgsrds _ V0  VR i
– 1 _V  V i
 0
(S)
C2D R

1  _V  V i
k 0
C2D R

rds R
+
1
C2D
R Vgs gmVgsrds k  V  V  slope
0 R

d 1 n
2

I 50pF
+ 
Rin V I + 0.5  0.2
– V
– 1
=
_50pF i # 0.7
2

KVL = 5.71 × 1020 F-2 V-1


Vgs + V = 0 ⇒ V = −Vgs Then option (a) is correct
V  g m rds Vgs  V
I  r x R  r R
39. For the solid S shown below, the value of
ds ds

g r VV
I  mr ds  R ### xdxdydz
ds

V  ds  R
s
r
I 1  g m rds (rounded off to two decimal places) is

ACE Engineering Publications Hyderabad  Delhi  Pune  Bhubaneswar  Bengaluru  Lucknow  Chennai  Vijayawada  Vizag  Tirupati  Kolkata  Ahmedabad
21 GATE_2020_Questions with Solutions

40. SPM(t) and SFM(t) as defined below, are the


z
phase modulated and the frequency modulated
1
waveforms, respectively, corresponding to the
1 y message signal m(t) shown in the figure.
0
SPM(t) = cos(1000pt + KPm(t))

and S FM ^ t h  cos c1000t  K f m ^  h d m


3
#
t
-3
x
where Kp is the phase deviation constant in

39. Ans: (2.25) radians/volt and Kf is the frequency deviation


Sol: From the figures, the limits of te integral are constant in radians/second/volt. If the highest
given by instantaneous frequencies of SPM(t) and SFM(t)
Z] Kp
]] x  0 to x  3 are same, then the value of the ratio K is _____
] f
lim its: [] y  0 to y  1 seconds. V
]]
]z  0 to z  1  y m(t)
\ 1 1-y 10

#> #* # xdz 4 dyHdx


3

I=
x=0 y=0 z=0

0
# > # x^z h dyHdx
3 1

= 1-y 1 3 6 7 t(sec)
x=0 y=0
0
40. Ans:2
Sol: highest instantaneous frequencies are same.
# > # x.`1  y jdyHdx
3 1

 i.e., fi, max, PM = fi, max, FM


x=0 y=0
 p
K dm (t)  f  K f m (t)
fc 2 dt max c 2
 c x m .d y  y n
2 3 2 1
max
2 0 dm (t)
2 0 = K f m (t) max
K p dt
9 max
= = 2.25 m(t)
4

z 10
z=1–y
z=1 1
z+y=1
1 3 6 7
1 y dm (t)
z=0
y=1 dt
y=0
z=0
5

z
1
x=0
y
1 −10

3 Kp . 5 = Kf . 10

x x=3 K p 10
= = 2 sec
Kf 5
ACE Engineering Publications Hyderabad  Delhi  Pune  Bhubaneswar  Bengaluru  Lucknow  Chennai  Vijayawada  Vizag  Tirupati  Kolkata  Ahmedabad
22 Electronics & Communication Engineering

ACE Engineering Publications Hyderabad  Delhi  Pune  Bhubaneswar  Bengaluru  Lucknow  Chennai  Vijayawada  Vizag  Tirupati  Kolkata  Ahmedabad
23 GATE_2020_Questions with Solutions

41. A pn junction solar cell of area 1.0 cm2, illuminated Step-I


uniformly with 100 mW cm-2, has the following
parameters: Efficiency = 15%, open circuit voltage Z
120∠–90° +

= 0.7V, fill factor = 0.8, and thickness = 200 mm. Z=(80–j35)Ω
The charge of an electron is 1.6 × 10-19 C. The I'
average optical generation rate (in cm-3 s-1) is
(a) 83.60 × 1019 (b) 1.04 × 1019
7120+  90 oA 8 j120B j120
(c) 5.57 × 1019 (d) 0.84 × 1019 I'  Z
  
80 j35 80  j35
41. Ans: (d)
Sol: A = 1cm2 Step-II
Pin (/cm2) = 100 mW/cm2
η = 0.15, Voc = 0.7, Fill factor = 0.8 I'' Z=(80–j35)Ω
Thickness = 200mm

=
Power (/cm2) =
100mW/cm2
5W/cm 3 120∠–30° + Z
200 # 10 -4

7120+  30 oA 7120+  30 oA
V I V I
 Pm m Fill factor = Vm I m
in oc SC
I''  Z

80  j35
Vm Im = η Pin
I  I'  I'' 
1 8 j120  103.92  j60B
`80  j35 j
Vm I m 0.75
= = # 0.8 = 1.339 A/cm
^ h
3
I SC

Voc Fill Factor 0 .7
`  103.92  j180 j
 207.34+120 o
o
Q I
I  t & Q  It  1.339A  s/cm 3 `80  j35 j 87.32+23.63
I = 2.380∠143.63°A
Q 1.339C/cm2
=Q nq = &n q = = 0.84 × 1019/cm3
1.6 # 10 -19 C
Optical generation rate = 0.84 × 1019/Cm3-s 43. For a 2-port network consisting of an ideal lossless
transformer, the parameter S21 (rounded off to
42. The current I in the given network is two decimal places) for a reference impedance of
10Ω, is _______.

120∠–90oV +
– Z 2 Port network
I
Z = (80−j35)Ω
2:1
120∠–30o V –+ Z
Port 1 Port 2

43. Ans: 0.8
(a) 2.38 ∠-23.63° A
Sol: For ideal Transformer,
(b) 2.38 ∠-96.37° A RS 2 V
SS n  1 2n WWW
(c) 2.38 ∠143.63° A
> 11 12H  SS n 1 n  12 WW
S S S  2 2
 W
(d) 0A S21 S22 S S 2 n 1 n WW
SS n2  1 n2  1 WW
42. Ans: (c) T X
2n 2#2
Sol: By applying super-position theorem ∵n = 2 ⇒ S12  S21  2   2   0.8
n 1 2 1
ACE Engineering Publications Hyderabad  Delhi  Pune  Bhubaneswar  Bengaluru  Lucknow  Chennai  Vijayawada  Vizag  Tirupati  Kolkata  Ahmedabad
24 Electronics & Communication Engineering

44. Consider the following closed loop control system (a) λ (b) 0.25λ
(c) 0.75λ (d) 0.5λ
+
R(s) C(s) G(s) Y(s) 45. Ans: (a)

Sol: Consider an infinitesimal electric dipole
^d, h, which is vertically placed at height ‘h’
where G ] s g  and C ] s g  K
1 s1 above an infinite grounded conductor.
.
^ 
s s 1 h s s  3h
^
If the steady state error for a unit ramp input is 0.1, The resultant electric field intensity is given by
jI o d, sin  -jbr
then the value of K is _____. Ei  4r e 2 cos _h cos  i; z $ 0
44. Ans: 30
=0 ;z<0
Sol:
+ (or)
R(s) C(s) jI o kd, sin  -jkr
e 2 cos ^kh cos h; z $ 0
G(s) Y(s)
– Ei  4r
= 0 ;z<0
where k (or) β is phase shift constant (or) wave
k ^s  1h
C]s gG]s g   k

s ^s  1h^s  3h s ^s  3h number.
 ^or h k  
2
1
ess (ramp) = k = 0.1
v
Given θmax = 60o
k v = lt s C ] s g G ] s g = lt s.
k  k

s"0 s"0 s ^s  3h 3 Field will have maxima,
1 3 when |cos(khcosθmax)| = 1
ess = k= k= 0.1
v ⇒ khcosθmax = π
k = 30 2

 h cos 60 
o

45. For an infinitesimally small dipole in free 2 1 


 .h. 2 
space, the electric field Eθ in the far field is
∴h=λ
c e m sin , where k  2 .
-jkr
proportional to r 
A vertical infinitesimally small electric dipole (or)
2E i
(δI << λ) is placed at a distance h( h > 0) above an =
2h 0
infinite ideal conducting plane, as shown in figure. jkI o d,e -jkr sin 
let P
The minimum value of ‘h’, for which one of the 4r
maxima in the far field radiation pattern occurs at 2 $ _ kh cos max i.  0
2h 2P cos
θ = 60° is _____.
−2pkcosθmaxsin(khcosθmax) = 0
z
sin(khcosθmax) = 0
I δl khcosθmax = nπ
assume n = 1
h θ kh cos60 = π
y 2 # 1 
 h. 2 
0
Infinite conduction plate ∴h=λ

ACE Engineering Publications Hyderabad  Delhi  Pune  Bhubaneswar  Bengaluru  Lucknow  Chennai  Vijayawada  Vizag  Tirupati  Kolkata  Ahmedabad
25 GATE_2020_Questions with Solutions

ACE Engineering Publications Hyderabad  Delhi  Pune  Bhubaneswar  Bengaluru  Lucknow  Chennai  Vijayawada  Vizag  Tirupati  Kolkata  Ahmedabad
26 Electronics & Communication Engineering

46. An ehhancement MOSFET of threshold voltage frequency fc is 1 MHz. The signal x(t) is passed
3 V is being used in the sample and hold circuit through a demodulator, as shown in the figure
given below. Assume that the substrate of the below. The output y(t) of the demodulator is
MOS device is connected to −10 V. If the input Ideal LPF with cut-off 510 Hz
voltage V1 lies between ±10 V, the minimum and 1
the maximum values of VG required for proper x(t) y(t)

sampling and holding respectively, are


–510 510 f(Hz)
V1 Vo Cos(2π(fc + 40)t)

(a) cos(540πt) (b) cos(920πt)


VG (c) cos(460πt) (d) cos(1000πt)
47. Ans:(b)

Sol:
(a) 13 V and −7V. (b) 10 V and −13 V.
LPF
(c) 10 V and −10 V. (d) 3 V and −3 V.
v(t)
46. Ans: (a)
Sol: x(t) y(t)
VS
Vi Vo

VGS cos2 π(fc + 40)t
+
VG −510 510

v (t)  m (t) cos (2fc t) cos (2 (fc  40) t)
VT = 3V
v (t)  4 cos (2500t) cos (2.106 t) cos (2 (106  40) t)
Vi = ±10V
v (t)  4 cos (2500t) 2 7cos (4106  240) t  cos (240t)A
1
During sampling (i.e when input ±10V)
MOSFET must be ON:
v (t)  2 cos (2.500t)7cos (2.40t)  cos (2 (2.106  40) tA
v (t)  cos (2.540t)  cos (2.460t)  cos (2 (2.106  540) t)
VGS > VT
 cos (2(2.106  460t))
VG − VS > VT [∵ VGS = VG − VS]
y (t)  cos (2460t)
VG > Vs + VT
y (t)  cos (920t)
VG > Vi + VT
when Vi = 10V ⇒ VG > 10V + 3V 48. A finite duration discrete time signals x[n] is
[when Vimax = 10V] obtained by sampling the continuous time signal
VG > 13V n
x(t) = cos(200πt) of sample instant t =
when Vi = −10V 400 ,
n = 0, 1, 2, ... 7. The 8-point discrete fourier
⇒ VG > Vi + VT [when Vimin = −10V] transform (DFT) of x(n) is defined as
X 5k? = / x ]ng .e -i 4 n, k = 0, 1, ...7
VG > −10 + 3V 7
rk

VG > −7V n=0
Which one of the following statements is
TRUE?
47. For the modulated signal x(t) = m(t)cos(2πfct), the (a) Only X(2) and X(6) are non-zero
message signal m(t) = 4cos(1000πt) and the carrier (b) Only X(3) and X(5) are non-zero

ACE Engineering Publications Hyderabad  Delhi  Pune  Bhubaneswar  Bengaluru  Lucknow  Chennai  Vijayawada  Vizag  Tirupati  Kolkata  Ahmedabad
27 GATE_2020_Questions with Solutions

(c) Only X(4) is non-zero 49. The transfer function of a stable discrete time LTI
] g
(d) All X( k ) are non-zero system is H ] z g = K z - a , where K and α
]z + 0.5g
48. Ans: (a) are real numbers. The value of α (rounded off to
Sol: x(t) = cos(200πt)
one decimal place) with a > 1, for which the
Samples
n x(n) magnitude response of the system is constant over
t=
400 all frequencies, is ____.

x ]nTsg  cos b 400 l & x ]ng  cos b 2 l
200n n 49. Ans: -2
Sol: H ]z g  K ^z  h , 
x(n) ={1, 0, −1, 0, 1, 0, −1, 0} z  0.5
1

X ]k g = / x ]n g W
N-1


H ]e j~g  1 6 
kn
N
n=0

/ x ]n g e For an all pass filter poles & zeros are reciprocal to


7
rkn
= -j 4

n=0
each other.
X ]4g  / x ]ng e -jrn  / x ]ng] 1gn
7 7

To get constant magnitude for all frequency pole


n=0 n=0
located at z = −0.5
X(4) = 1 − 1 + 1 − 1 = 0 1 
∴ zero is located at =  2
X ]2g  / x ]ng e -j 2  1  1  1  1  4 k ]z  2g
7
rn 0.5

n=0 H ]z g  
z 0.5
As X(k) = X*(N − K) ∴ α = −2
X(2) = X*(8−2) = X*(6)
Only X(2) & (6) are non zero. 50. The base of an npn BJT T1 has a linear doping
(or) profile NB(x) as shown below. The base of another
when we look at x(n), it contains only even non BJT T2 has a uniform doping NB of 1017 cm−3.
samples. If we assume z(n) = {1−1 1−1} All other parameters are identical for both the
RS ] gVW RS VR V R V devices. Assuming that the hole density profile is
SSZ 0 WW SS1 1 1 1 WWWSSS 1 WWW SSS0WWW
SZ ]1 gWW SS1  j  1 j WWSS 1WW SS0WW the same as that of doping, the common-emitter
Z(k) = SSS WS WS W  S W
SSZ ]2gWWW SSS1  1 1  1WWWSSS 1 WWW SSS4WWW current gain of T2 is
SSZ ]3 gWW SS1 j  1  j WWSS 1WW SS0WW
T X T XT X T X
1017(cm−3)

We know zero interpolation in time domain NB(x)
corresponds to replication of DFT spectrum
=
x ]ng z=b n l DFT X ]kg " Z ]kg, Z ]kg,
2
= {0,0,4,0,0,0,4,0}
1014(cm−3)
(or) Emitter Base Collector
cos c N k 0 n m ) 2 9 _ k  k 0 i   _ k  k 0 iC
2 N

0 W

cos c 2 m  cos c 8 2n m ) 2 8 ^k  2 h   ^k  2 hB
n 2 8 (a) approximately 0.3 times that of T1.

(b) approximately 2.0 times that of T1.
So, non zero samples are -2, 2. From periodicity (c) approximately 0.7 times that of T1.
property non zero samples are -2+8, 2 = 6,2. (d) approximately 2.5 times that of T1.

ACE Engineering Publications Hyderabad  Delhi  Pune  Bhubaneswar  Bengaluru  Lucknow  Chennai  Vijayawada  Vizag  Tirupati  Kolkata  Ahmedabad
28 Electronics & Communication Engineering

50. Ans: (b) 51. Ans: (d)


Sol:  T2  Area under the carrier profile of base of T1 Sol: R
 T1 Area under the carrier profile of base of T2
C

R 0V
1017(cm−3) –
NB(x) + V0
Vin +
2R
– 1/2Cs


No current flows through 2C and 2R
1014(cm−3)
Emitter Base Collector KCL
Vin  0 0  0 0  0 0  Vo 0  Vo
0     
W R 1/2sC 2R 0 1/sC R
T1

 Vo ;  sCE
Vin 1
R R
Vo 1
  1
NB(x) Vin 1  sCR 1  s
1 1
3dB    RC
1017(cm−3)
1 1
f3dB  2RC   79.58Hz
2 ^2k h_1 i
Emitter Base Collector
52. X(ω) is Fourier transform of x(t) shown below.
0 W 3
X ]~g 2 d~ (rounded off to
T2
The value of #
 T2 1017 # W -3
 two decimal places) is ______.
 T1 1 _ 17  14 i  14
2 10 10 W 10 W x(t)
 1017
1 _ 17  14 i  14 - 2 3
2 10 10 10
⇒ bT2 = 2bT1

51. The components in the circuit given below are t


–3 –2 –1 0 1 2 3 4
ideal. If R = 2 kΩ and C = 1 µF, the −3 dB cut-off
frequency of the circuit in Hz is 52. Ans: 58.64
Sol: From parseval’s theorem
R
X ]g 2 d  2 x ] t g 2 dt
3 3

C
# #
-3 -3

Vi(jω) R

V0(jω)
+
2R
2C

(a) 14.92 (b) 59.68
(c) 34.46 (d) 79.58

ACE Engineering Publications Hyderabad  Delhi  Pune  Bhubaneswar  Bengaluru  Lucknow  Chennai  Vijayawada  Vizag  Tirupati  Kolkata  Ahmedabad
29 GATE_2020_Questions with Solutions

x(t) P+Q+R = −11


= 2’s complement of +11
3 = 2’s complement of 001011
= 110101
1
Thus P+Q+R in 6-bit 2’s complement representation
t is 110101.
–1 0 1 2 3
54. The band diagram of a p-type semiconductor
Because of symmetry of the signal w.r.t to t = 1 with a band-gap of 1 eV is shown, using this
x ] t g 2 dtH
3

X ]g d  2 >
+3

# 2
# semiconductor, a MOS capacitor having VTH of
-3 -1
−0.16 V, Clox of 100 nF/cm2 and a metal work
0 1

= 2 ]2g> # ^t  1h dt  # ^2t  1h dtH function of 3.87 eV is fabricated. There is no


2 2

-1 0 charge within the oxide. If the voltage across the


capacitor is VTH, the magnitude of depletion charge
0 1

2 ]2g> # ^t  1h dt  # ^4t  4t  1h dtH


2 2

-1 0 per unit area (in C/cm2) is


Vacuum level
^t  1h3 0
= 4;; E  : 4t  4t  tD E
3 2 1
4 eV
3 -1 3 2 0
EC
= 4 : 3  3  2  1D  4 : 3  3D 0.5 eV
1 4 5
Ei

=
56 EFs
3 0.2 eV
EV
= 58.64
(a) 1.41 × 10−8 (b) 1.70 × 10−8
(c) 0.52 × 10−8 (d) 0.93 × 10−8
53. P, Q, and R are the decimal integers corresponding
54. Ans: (b)
to the 4-bit binary number 1100 considered
Sol: Vaccum level
in signed magnitude, 1’s complement and 2’s
complement representations, respectively. The 4eV
6-bit 2’s complement representation of (P + Q + R) φS = 4.8eV
EC
is
0.5eV
(a) 111101
(b) 110101 Ei
0.3eV qφF
(c) 110010 EF
0.2eV
(d) 111001 E
S
V
φS = 4eV+0.5eV+0.3eV=4.8eV
53. Ans: (b)
Sol: P = The decimal value of sign magnitude no 1100
VTH = -0.16V, Cox =100nF/cm2
= −4
φm= 3.87 eV
Q = The decimal value of 1’s complement no 1100
Q dinv
= −3 VT  VFB  C  t
ox
R = The decimal value of 1’s complement no 1100
VFB = φms (a No oxide charge)
= −4
VFB = φms = φm - φs = 3.87 - 4.8 = - 0.93
Thus P+Q+R = (−4) + (−3) + (−4)
ACE Engineering Publications Hyderabad  Delhi  Pune  Bhubaneswar  Bengaluru  Lucknow  Chennai  Vijayawada  Vizag  Tirupati  Kolkata  Ahmedabad
30 Electronics & Communication Engineering

φt = 2 φF, φF = 0.3V R2 ↔ R3
φt = 2 × 0.3 = 0.6V RS V
SS1 2 b1 WWW
 0.16  _  0.93 i  dinv  0.6
Q
SS0 1 b − 3b WW
100n _ A B i + SS 3 1
W
SS0 0 b2 − 2b1WWW
|Qdinv| = 0.17 × 100 × 10-9 = 17 × 10-9 SS0 3 b − 3b WW
4 1
= 1.7 × 10-8C/cm2 T X
R4 → R4 − 3R2
55. Consider the following system of linear equation. RS VW
x1 + 2x2 = b1 ; 2x1 + 4x2 = b2 ; 3x1 + 7x2 = b3 ; SS1 2 b1 W
SS0 1 b  3b WWW
3x1 + 9x2 = b4 _ A B i + SS 3 1
W
SS0 0 b2  2b1 WWW
Which one of the following conditions ensures that S0 0 6b  3b  b WW
S 1 3 4
a solution exists for the above system? T X
Here, ρ(A) = 2
(a) b2 = 2b1 and 3b1 − 6b3 + b4 = 0
To have a solution, ρ(A) = 2 = ρ(A|B)
(b) b3 = 2b1 and 3b1 − 6b3 + b4 = 0
if b2 − 2b1 = 0 and 6b1 − 3b3 + b4 = 0 then
(c) b3 = 2b1 and 6b1 − 3b3 + b4 = 0
ρ(A|B) = 2
(d) b2 = 2b1 and 6b1 − 3b3 + b4 = 0
∴ solution exists if b2 = 2b1
55. Ans: (d)
and 6b1 −3b3 + b4 = 0
Sol: Consider augmented matrix
RS V Hence, option (d) is correct.
SS1 2 b1WWW
SS2 4 b WW
_ A B i = SS 2
W
SS3 7 b 3WWW
SS3 9 b WW
4
T X
R2 → R2 − 2R1; R3 → R3 − 3R1; R4 → R4 − 3R1
RS V
SS1 2 b1 WWW
SS0 0 b − 2b WW
_ A B i + SS 2 1
W
SS0 1 b 3 − 3b1WWW
SS0 3 b − 3b WW
4 1
T X

ACE Engineering Publications Hyderabad  Delhi  Pune  Bhubaneswar  Bengaluru  Lucknow  Chennai  Vijayawada  Vizag  Tirupati  Kolkata  Ahmedabad
31 GATE_2020_Questions with Solutions

ACE Engineering Publications Hyderabad  Delhi  Pune  Bhubaneswar  Bengaluru  Lucknow  Chennai  Vijayawada  Vizag  Tirupati  Kolkata  Ahmedabad
32 Electronics & Communication Engineering

ACE Engineering Publications Hyderabad  Delhi  Pune  Bhubaneswar  Bengaluru  Lucknow  Chennai  Vijayawada  Vizag  Tirupati  Kolkata  Ahmedabad

You might also like